Showing composition of functions are uniformly continuous

Click For Summary
The discussion focuses on proving that the sum of two uniformly continuous functions, f and g, on an interval I is also uniformly continuous. The proof begins by establishing the existence of δ values for both functions based on the definition of uniform continuity. Using the triangle inequality, it shows that the sum of the functions meets the uniform continuity criteria. Minor suggestions for improvement include using ε/2 for the inequalities and simplifying the choice of δ. The original poster acknowledges the corrections and confirms their intention to prove the sum, not the composition, of the functions.
k3k3
Messages
76
Reaction score
0
Showing the sum of functions are uniformly continuous

Homework Statement


Suppose f and g are uniformly continuous on an interval I. Prove f + g are uniformly continuous on I.

Homework Equations





The Attempt at a Solution



Let ε >0

By definition, since f and g are uniformly continuous on I, there exists a \delta_1 such that |f(y)-f(x)| < ε for all x,y in I that satisfy |x-y| < \delta_1

Similarly, for g, there exists a \delta_2 such that |g(y)-g(x)| < ε for all |y-x| < \delta_2

Then, for all x,y in I, |f(y)+g(y)-(f(x)+g(x))| ≤ |f(y)-f(x)|+|g(y)-g(x)| by the triangle inequality. This implies |f(y)+g(y)-(f(x)+g(x))| < 2ε

Choose \delta=min{\delta_1,\delta_2} * 1/2

Then |f(y)+g(y)-(f(x)+g(x))| < 2ε for all x,y in I that satisfy |y-x| < \delta

∴f+g is uniformly continuous on I.

Is this correct?
 
Last edited:
Physics news on Phys.org
Yes, but a couple of minor changes would make it slightly nicer. For your first two inequalites use ##\frac \epsilon 2## so it will only add to ##\epsilon## at the end. And choosing ##\delta##, there is no need to multiply by 1/2. Finally, where you have:

|f(y)+g(y)-(f(x)+g(x))| ≤ |f(y)-f(x)|+|g(y)-g(x)| by the triangle inequality. This implies |f(y)+g(y)-(f(x)+g(x))| < 2ε

you could instead just write

|f(y)+g(y)-(f(x)+g(x))| ≤ |f(y)-f(x)|+|g(y)-g(x)| < ε/2+ε/2 =ε.

Nice work. And, by the way, you have shown the sum of two u.c. functions is u.c. If you really meant the composition, you did the wrong problem :rolleyes:
 
Last edited:
Oops! I did mean the sum. I am not sure why I wrote that... Thank you for the suggestions. I will rewrite it to make it look nicer!
 
Question: A clock's minute hand has length 4 and its hour hand has length 3. What is the distance between the tips at the moment when it is increasing most rapidly?(Putnam Exam Question) Answer: Making assumption that both the hands moves at constant angular velocities, the answer is ## \sqrt{7} .## But don't you think this assumption is somewhat doubtful and wrong?

Similar threads

  • · Replies 5 ·
Replies
5
Views
1K
  • · Replies 5 ·
Replies
5
Views
2K
Replies
32
Views
3K
Replies
5
Views
2K
  • · Replies 40 ·
2
Replies
40
Views
4K
  • · Replies 8 ·
Replies
8
Views
2K
Replies
22
Views
2K
  • · Replies 3 ·
Replies
3
Views
1K
  • · Replies 6 ·
Replies
6
Views
2K
  • · Replies 27 ·
Replies
27
Views
2K